How to expand the span of a vector set in linear algebra?

Click For Summary
To expand the span of a vector set in linear algebra, it's essential to add a third vector to a 3x2 matrix, as two vectors alone cannot span R3. First, check if the existing two vectors are linearly independent; if they are, find a third vector that is also linearly independent from them. If the two vectors are dependent, select two additional vectors that, along with one of the original vectors, form a linearly independent set. Construct a matrix with these vectors and ensure the determinant is non-zero to confirm they span R3. This approach guarantees the expanded set will cover the entire three-dimensional space.
wishnight
Messages
2
Reaction score
0

Homework Statement


I do understand that in matrix 3x2, the set of vector doesn't span of R3. What should I do to make the set of vector span of R3.


Homework Equations





The Attempt at a Solution


I think adding one more set of vector is the best idea. So, if I can add one more set of vector to make them span of R3, how to find the third set of vector. thanks
 
Physics news on Phys.org
I think you need to state the question more clearly. In a 3x2 matrix the columns don't span R^3. The three standard vectors (1,0,0), (0,1,0), (0,0,1) span R^3. I you want to make a matrix of which the colums span R^3 put these in a matrix.

If you want to use the two vectors you already got from the matrix:
- first check to see if the two you have are linearly independent.
i.e. whether one is a mulitple of the other (in the case of 2 vectors)
- If they are find a third vector that is also linearly independt of both the vectors (together)
- I they aren't find two more that, together with one of the vectors you already had,
are linearly independent of each other.

Then put them in a matrix.
 
I mean if we got 2 vector a = (1,2,1) and b = (1,3,1). Then I need to create one more vector that will make the set of vectors span of R3
 
There are many way's to do this. One way is to put these to vectors and (a,b,c) in a matrix. Calculate the determinant then choose a,b,c such that the determinant is not 0. Any such a,b,c will do
 
Question: A clock's minute hand has length 4 and its hour hand has length 3. What is the distance between the tips at the moment when it is increasing most rapidly?(Putnam Exam Question) Answer: Making assumption that both the hands moves at constant angular velocities, the answer is ## \sqrt{7} .## But don't you think this assumption is somewhat doubtful and wrong?

Similar threads

Replies
15
Views
2K
  • · Replies 3 ·
Replies
3
Views
3K
  • · Replies 6 ·
Replies
6
Views
2K
  • · Replies 4 ·
Replies
4
Views
2K
  • · Replies 10 ·
Replies
10
Views
3K
Replies
4
Views
1K
  • · Replies 15 ·
Replies
15
Views
3K
Replies
4
Views
2K
  • · Replies 30 ·
2
Replies
30
Views
1K
  • · Replies 3 ·
Replies
3
Views
2K